Difference between revisions of "2020 AMC 12A Problems/Problem 1"

(Problem)
(Solution)
Line 6: Line 6:
  
 
==Solution==
 
==Solution==
 +
 +
If Carlos took 70% of the pie, 30% must be remaining.
  
 
==See Also==
 
==See Also==

Revision as of 10:30, 1 February 2020

Problem

Carlos took $70\%$ of a whole pie. Maria took one third of the remainder. What portion of the whole pie was left?

$\textbf{(A)}\ 10\%\qquad\textbf{(B)}\ 15\%\qquad\textbf{(C)}\ 20\%\qquad\textbf{(D)}\ 30\%\qquad\textbf{(E)}\ 35\%$

Solution

If Carlos took 70% of the pie, 30% must be remaining.

See Also

2020 AMC 12A (ProblemsAnswer KeyResources)
Preceded by
First Problem
Followed by
Problem 2
1 2 3 4 5 6 7 8 9 10 11 12 13 14 15 16 17 18 19 20 21 22 23 24 25
All AMC 12 Problems and Solutions

The problems on this page are copyrighted by the Mathematical Association of America's American Mathematics Competitions. AMC logo.png